complex# prob (1 Viewer)

Faera

Member
Joined
Mar 12, 2004
Messages
48
Location
Sydney
Yup, I'm stuck, yet again:

(i) State a condition for z, w, v (all complex) to form the vertices of an equilateral triangle on the Argand Diagram.

(ii) If z = 1 + 3i, find a complex number w, so that z, w and the origin O form the vertices of an equilateral triangle.

Thanks for taking a look, and any help would be good :)


p.s. Cake for whoever explains it to me first!!
;)
 

Grey Council

Legend
Joined
Oct 14, 2003
Messages
1,426
Gender
Male
HSC
2004
first one:
look below at Affinities reply OR:
(z-w)/(w-v) = (w-v)/(v-z) = (v-z)/(z-w) = cis(+-pi/3)

second one:
cos(arctan3+pi/3) + isin(arctan3+pi/3)
expand that all out, you'll get exact form.
i hate trig. i'm not even gonna try. lol

I give up. no cake for me, i s'pose. berloody, cm_tutor will get it again. meh
 
Last edited:

Affinity

Active Member
Joined
Jun 9, 2003
Messages
2,062
Location
Oslo
Gender
Undisclosed
HSC
2003
(i) notice how it says 'a condition'? so a perfectly legitimate answer would be z,w,v are distinct cube roots of unit.

the general condition would be (z-w),(w-v),(v-z) are k, xk,x^2k in any order,where x is a complex cube root of unity

ii) rotate 1+3i clockwise or anticlockwise 60 degrees about the origin.. which is same as multiplying by cis(Pi/6) and cis(-Pi/6)

will leave you to do it
 

CM_Tutor

Moderator
Moderator
Joined
Mar 11, 2004
Messages
2,644
Gender
Male
HSC
N/A
Another general condition would be |z - w| = |w - v| = |v - w|

Affinity, I think you mean pi / 3, not pi / 6.
 

Faera

Member
Joined
Mar 12, 2004
Messages
48
Location
Sydney
Hmm...
Grey_C, I don't really get what you're saying... perhaps i'll get you to explain it to me over msn later on... -.-'

Affinity, what you're saying's true, but then what if the equilateral trianlge formed isnt actually formed with the origin as center? Because for part (ii), one of the vertices is actually O...

CM- by |z - w| = |w - v| = |v - w|, do you mean |z - w| = |w - v| = |v - z| ?
 

CM_Tutor

Moderator
Moderator
Joined
Mar 11, 2004
Messages
2,644
Gender
Male
HSC
N/A
Originally posted by Faera
CM- by |z - w| = |w - v| = |v - w|, do you mean |z - w| = |w - v| = |v - z| ?
Yes, you're right, I've made a typo on the last term. Oops. :)

As for the other part, there are two possible points, which I get as (1 - 3 * sqrt(3)) / 2 + (3 + sqrt(3))i / 2 or
(1 + 3 * sqrt(3)) / 2 + (3 - sqrt(3))i / 2
 

Grey Council

Legend
Joined
Oct 14, 2003
Messages
1,426
Gender
Male
HSC
2004
Originally posted by Faera
Hmm...
Grey_C, I don't really get what you're saying... perhaps i'll get you to explain it to me over msn later on... -.-'

Affinity, what you're saying's true, but then what if the equilateral trianlge formed isnt actually formed with the origin as center? Because for part (ii), one of the vertices is actually O...

CM- by |z - w| = |w - v| = |v - w|, do you mean |z - w| = |w - v| = |v - z| ?
hrm, your rotating the LINE 1+3i by 60 degrees. ^_^ your sliding the line. rotating about the origin. hope you know what i mean.
 

Users Who Are Viewing This Thread (Users: 0, Guests: 1)

Top